LSAT and Law School Admissions Forum

Get expert LSAT preparation and law school admissions advice from PowerScore Test Preparation.

User avatar
 Gorrochategui
  • Posts: 3
  • Joined: Apr 12, 2023
|
#100830
For this question, I do understand why answer choice A is correct: "Presumes without providing any justification, that the surest way of increasing the overall corrctness of one's beliefs must not hinder one's ability to survive."

I however, chose the very popular incorrect answer choice: B: "Neglects the possibility that even while following the statician's rule, one might also accept new beliefs when new evidence is presented."

The authors conclusion is the the staticians claim must be incorrect because "we need many beliefs in order to survive."
The conclusion introduces an idea that had not been previously mentioned, which is one of the two ways in which a conclusion can be flawed.

When doing descriptive weakening questions, I accept ALL the information in the stimulus ass true? In this case, the statician specifically mentions that one must never change the set of one's beliefs, except by rejecting some of our beliefs in that set when presented with evidence; is answer choice B wrong because it is going against what was said in the stimulus? If one must never change one's set of beliefs, than clearly we must not accept new beliefs when presented with new evidence as answer choice B says.

Help please.
 Luke Haqq
PowerScore Staff
  • PowerScore Staff
  • Posts: 751
  • Joined: Apr 26, 2012
|
#100837
Hi Gorrochategui!

You correctly identify some key elements of the stimulus, such as the conclusion as well as noting that the conclusion brings in a new element.

Answer choice (B) states that the flaw is that the argument "neglects the possibility that even while following the statisticians’ rule, one might also accept new beliefs when presented with some kinds of evidence." One reason this seems incorrect is that it doesn't address the new element in the conclusion of needing beliefs to survive. If a new element sticks out to you when reading through the stimulus, you'd want to be looking out for an answer choice that addresses it, as (A) does.

A possible reason that (B) might have been tempting is that it does address material in the stimulus, specifically in the second sentence. However, even if the author accepted that people might accept new beliefs, we're still left wondering how the mentioned rule relates to one's ability to survive.
 mollylynch
  • Posts: 62
  • Joined: Jul 21, 2023
|
#103832
Can you explain what A means by "surest way of increasing the overall correctness of the total set of one's beliefs must not hinder one's ability to survive"?
 Luke Haqq
PowerScore Staff
  • PowerScore Staff
  • Posts: 751
  • Joined: Apr 26, 2012
|
#104172
Hi mollylynch!

The conclusion of this stimulus is, "Since we need many beliefs in order to survive, the statisticians’ claim must be mistaken." Hopefully when reading this conclusion, you noticed that there's a new element brought in, as the concept of survival wasn't mentioned in any of the previous sentences.

Answer choice (A) suggests that the author of this stimulus is making a connection between the first sentence ("Some statisticians claim that the surest way to increase the overall correctness of the total set of one’s beliefs...") and the final sentence. Effectively, the author is assuming that the "surest way to increase the overall correctness of the total set of one's beliefs" mentioned in the first sentence (i.e., the best way to make one's set of beliefs as correct as possible) can't hinder one's abilities to survive, mentioned in the final sentence.

Get the most out of your LSAT Prep Plus subscription.

Analyze and track your performance with our Testing and Analytics Package.